Query failed: connection to 172.31.3.4:9312 failed (errno=111, msg=Connection refused). PT147.S4.Q12 - West: Of our company's three quality control inspectors - 7Sage Forum

PT147.S4.Q12 - West: Of our company's three quality control inspectors

name_namename_name Alum Member
edited August 3 in Logical Reasoning 5 karma

Can someone explain why E and not B is the correct answer? I am not totally convinced by the explanation given here. This is because the idea that the work is split equally is not explicitly used as "one of the argument's presuppositions." However, the relevance of West's conclusion to the premises is directly implicated as he clearly does not use the relevant evidence (the proportion of correct decisions made by the quality control workers). Any help would be appreciated.

Comments

  • evanjones24evanjones24 Core Member
    3 karma

    I agree that JY's explanation for why B is wrong is lack-luster at best lol. But I think you can see that B is wrong by considering what it means for something to not be "relevant". It is not that Haynes being the worst inspector is entirely irrelevant from the concept of how many defective products were inspected by him. If every defective product returned was inspected by Haynes, this would be pretty strong evidence that Haynes is the worst inspector.

    Young instead responds by challenging the pressuposition that because >half were inspected by Haynes, Haynes overlooks more defective products (as a proportion of those inspected) relative to the other 2 inspectors. I don't like JY's explanation because he imagines that the three inspectors each inspect 1/3. But this is not technically the presupposition in West's argument. The presupposition is simply that Haynes' products being >half of the defective ones reflects some disproportionality (i.e. he inspects less than half of the products). This is what Young denies in his response.

    B seems tempting cause it feels like this makes the conclusion "irrelevant" to the premises–and in a sense it does (as does basically every challenge to an argument). But it does so by denying the pressupositon which makes E a much better answer. For B to be the best answer, it would require a much more immediate dismissal of the conclusion as entirely disconnected from the premises.

  • LSATenjoyer180LSATenjoyer180 Core Member
    26 karma

    For B to be correct it would mean the conclusion is a non sequitur. For instance if the conclusion was about Haynes being unethical just using those premises about the relative amount of products returned that were inspected by Haynes.

    Also presuppositions, another word for assumptions, by definition are never explicitly stated. That West inspects an equal amount of product or less is a necessary assumption for the argument to be valid because if its not: West inspects 95% of the products and accounts for half of all returns while the other two inspect 5% and account for the other half then West is clearly a better inspector. C is just describing Young saying the necessary assumption/presupposition is wrong.

  • beyondpossumbeyondpossum Live Member
    edited August 3 89 karma

    Think of what the stim would have to look like for B to be correct. West would have to say something, and Young would have to reply something like "Well, that very well may be true, but is irrelevant to the task or debate at hand." But that's not how Young goes about things. Young is in fact interested in denying what West is insinuating.

    E is itself not perfect, because it's not necessarily that Young is denying a presupposition of the argument; he may instead be calling out a statistical, non-sequitor fallacy, because he thinks West is too dumb to be encoding the needed lemma for the argument to go through. In any case, the answer clearly isn't C, because the needed lemma is not explicitly stated.

    Edit: I see that the commenters above are interpreting B differently than me (In fact, they interpret it differently from each other. This might actually indicate that this question wasn't vetted very well if three different LSAT taker has come to three different interpretations about the same short sentence.) The first commenter says that B is alleging that Young's conclusion has nothing to do with his premises. This would still make B false; after all, the premises do seem to have something to do with Young's conclusion in that they tend to support his conclusion--they're just missing a key extra premise. The second commentor interprets B as alleging a non sequitur. I hope this is not the case because then B is a perfectly fine answer choice and the question is in fact broken.

Sign In or Register to comment.